+0  
 
+2
988
7
avatar+2862 

Henry decides one morning to do a workout and walks \(\frac{3}{4}\) of the way from his home to his gym. The gym \(2\) kilometers away from Henry's home. At that point, he changes his mind and walks \(\frac{3}{4}\) of the distance from there back toward the gym. If Henry keeps changing his mind when he has walked \(\frac{3}{4}\)of the distance toward either the gym or home from the point where he last changed his mind, he will get very close to walking back and forth between a point \(A\) kilometers from home and a point \(B\) kilometers from home. What is \(|A-B|\) ?

 

A. \(\frac{2}{3}\)

 

B.​ \(1\)

 

C. \(\frac{6}{5}\)

 

D. \(\frac{5}{4}\)

 

E. \(\frac{3}{2}\)

 Feb 15, 2019
 #1
avatar+53 
+1

This is an AMC 10B problem. You probably shouldn't post it until everyone has taken the test (maybe next week?). By the way, I was also stuck on this problem.

 Feb 15, 2019
 #2
avatar+98 
0

ouch this is good i didnt answer this isnt it!cheeky

StarStrike  Feb 15, 2019
 #3
avatar+53 
0

I know its a AMC 10B problem because I also took the AMC 10B

Mathcounts24  Feb 15, 2019
 #4
avatar+4609 
0

the amc10b has passed just to let you know.

tertre  Feb 15, 2019
 #5
avatar+53 
0

ya I figured that out like right after I posted the second message

Mathcounts24  Feb 15, 2019
 #6
avatar+746 
0

what is an amc 10B test? Im just wondering.

ilovepuppies1880  Feb 15, 2019
 #7
avatar+53 
0

I posted that on the AIME post, look at that

Mathcounts24  Feb 18, 2019

0 Online Users